Finding a Sequence with Limit Set [0,1]

Click For Summary
A sequence that has a subsequential limit set of the interval [0,1] can be constructed using the absolute value of the sine function, |sin n|, as it densely covers the interval. To prove this, one must show that for any point in [0,1] and given an epsilon>0, there are infinitely many sequence members within that neighborhood. An alternative approach involves using n*sqrt(2) mod 1, which also densely populates the interval and may simplify the proof. The discussion highlights the connection between the closure of rational numbers in [0,1] and the interval itself, emphasizing the importance of cluster points. Overall, the conversation revolves around finding rigorous proofs for sequences that meet the specified limit criteria.
PingPong
Messages
61
Reaction score
0

Homework Statement


Find a sequence whose set of subsequential limits is the interval [0,1].


Homework Equations


If the sequence does not repeat itself, then any subsequential limit is a cluster point.

The Attempt at a Solution



I've an idea that |\sin n| is a solution to this, but I'm not sure how to prove this.

What I need to show is that, since the sequence I've chosen is non-repeating, then every point in the interval [0,1] is a cluster point. That is, if I take any number in [0,1] and am given an epsilon>0, can I always find an infinite amount of members in the sequence in an epsilon neighborhood around the number? This seems to intuitively work since |\sin n| is all over the interval [0,1] (though not necessarily touching every point in the interval), but can this argument be made more rigorous? Any suggestions?
 
Physics news on Phys.org
What can the rationals in [0,1] do for you?
 
|sin(n)| does almost certainly work. But I don't know an obvious proof of this. Maybe think about an easier example though. How about n*sqrt(2) mod 1? That should be 'all over', too, and maybe easier to deal with. Sorry, but I was just going zzzzz and found your post, so I can't give hints right now.
 
morphism said:
What can the rationals in [0,1] do for you?

Works also, and even easier. Nice.
 
Thanks Dick and Morphism. I feel really silly now - I was trying to make it much too complicated. Obviously, since the closure of Q\cap [0,1] is again [0,1], then the subsequential limits of a sequence of rational numbers in that interval (say, in the order given by the Cantor diagonalization process, throwing away those outside the interval) is the interval itself.

As a side note, in my search for information on cluster points, I stumbled upon an article (on JStor, which my university subscribes to, so I won't post the url since most people won't be able to access) that gives a proof that the cluster points of sin n is the interval [-1,1].
 
Question: A clock's minute hand has length 4 and its hour hand has length 3. What is the distance between the tips at the moment when it is increasing most rapidly?(Putnam Exam Question) Answer: Making assumption that both the hands moves at constant angular velocities, the answer is ## \sqrt{7} .## But don't you think this assumption is somewhat doubtful and wrong?

Similar threads

  • · Replies 5 ·
Replies
5
Views
2K
  • · Replies 4 ·
Replies
4
Views
2K
Replies
8
Views
2K
  • · Replies 11 ·
Replies
11
Views
3K
Replies
7
Views
2K
  • · Replies 1 ·
Replies
1
Views
1K
  • · Replies 5 ·
Replies
5
Views
2K
  • · Replies 16 ·
Replies
16
Views
3K
  • · Replies 7 ·
Replies
7
Views
2K
  • · Replies 2 ·
Replies
2
Views
1K